The cost price of a box and a pen isRs 120 The box is sold at lo% profit and the pen is sold at 10% loss. If the selling price of the box is Rs 52 more than that of the pen, calculate the profit or loss percent in a whole.​

Answers

Answer 1

Answer:

profit of 3.33%

Step-by-step explanation:

Answer 2

Answer:

3.33%

Step-by-step explanation:

Cos of Pen = pCos of Box = bb + p = 1201.1b - 0.9p = 52

Solve the system by substitution:

p = 120 - b1.1b - 0.9(120 - b) = 521.1b + 0.9b = 52 + 1082b = 160b = 160/2b = 80

Find p:

p = 120 - 80 = 40

Total profit or loss:

80*1.1 + 40*0.9 - 120 = 4 (profit as positive)

Profit percent:

4/120*100% = 3.33%

Related Questions

the best way to learn math formulas

Answers

Writing down the formulas on charts and pasting it in your room,by seeing this daily it helps to memorize the formulas.

Saying the formulas louder also helps to memorize the formula.

Watching videos related to maths formulas and equations helps to remember the formulas easier.

Doing many problems regularly will helps you to remember the formulas.

lastly study to Understand The Formula not to memorize

ABCD is a quadrilateral, P Q R and S divide AB, CB, CD and AD in ratio 1:3 respectively. Prove that a//m. (Please hurry up, I will mark you as Brainliest) ​

Answers

Two lines are parallel if they are equidistant from one another. The following proves [tex]a || m[/tex]

[tex]Ratio = 1:3[/tex] means that:

[tex]BP=3PA[/tex]

[tex]BQ=3QC[/tex]

To prove that [tex]a || m[/tex] is to prove that side length SR and side length PQ are parallel because:

[tex]SR = a\\ PQ = m[/tex] --------- See attachment for quadrilateral

Considering [tex]\triangle ABC[/tex], we have: [tex]BP=3PA[/tex]

Divide both sides by PA

[tex]\frac{BP}{PA} = \frac{3PA}{PA}[/tex]

[tex]\frac{BP}{PA} = 3[/tex]

Similarly: [tex]BQ=3QC[/tex]

Divide both sides by QC

[tex]\frac{BQ}{QC} = \frac{3BQ}{QC}[/tex]

[tex]\frac{BQ}{QC} = 3[/tex]

So, we have:

[tex]\frac{BP}{PA} = \frac{BQ}{QC} = 3[/tex]

Express as ratios

[tex]BP:PA = BQ:QC = 3[/tex]

[tex]BP:PA \to[/tex] means point P divided side length BA into 3:1

[tex]BQ:QC \to[/tex] means point Q divided side length BC into 3:1

Thales Theorem states that: A line that divides any two sides of a triangle in the same ratio is parallel to the third side.

The third side of [tex]\triangle ABC[/tex] is [tex]AC[/tex].

This means that: [tex]PQ[/tex] is parallel to [tex]AC[/tex]

Since [tex]AC[/tex] is parallel to [tex]SR[/tex], then [tex]PQ[/tex] is parallel to [tex]SR[/tex]

i.e. [tex]SR || PQ[/tex]

Recall that:

[tex]SR = a\\ PQ = m[/tex]

Hence:

[tex]a || m[/tex] has been proved.

Read more about proof of parallel sides at:

https://brainly.com/question/2510808

A cylindrical paint can has a diameter of 12 centimeters and height of centimetrs which is closest to the volume of the paint can in cubic centimeters

Answers

Answer:

The correct answer is "1808.64 cm³".

Step-by-step explanation:

Seems that the given query is incomplete. Below find the attachment of the complete problem.

Given:

Diameter,

d = 12 cm

Radius,

r = [tex]\frac{d}{2}[/tex]

 = [tex]\frac{12}{2}[/tex]

 = [tex]6 \ cm[/tex]

Height,

h = 16 cm

As we know,

The volume of cylinder is:

= [tex]\pi r^2 h[/tex]

By substituting the values, we get

= [tex]3.16\times (6)^2\times 16[/tex]

= [tex]3.14\times 36\times 16[/tex]

= [tex]1808.64 \ cm^3[/tex]

What is the exponential form of the logarithmic equation?
5 = logo.9 0.59049

Answers

^ means to the power of or exponent

logb(x) = y is b^y = x

Answer:

[tex]0.9^5=0.59049[/tex]

Step-by-step explanation:

By definition, we have:

[tex]log_ab=c\implies a^c=b[/tex]

Therefore, given [tex]log_{0.9}(0.59049)=5[/tex], assign:

[tex]a=0.9[/tex] [tex]b=0.59049[/tex] [tex]c=5[/tex]

Rewrite:

[tex]\boxed{0.9^5=0.59049}[/tex]

given m||n, find the value of x

Answers

Answer:

x=16

Step-by-step explanation:

The angles indicated are alternate exterior angles and alternate exterior angles are equal when the lines are parallel

5x+28 = 7x-4

Subtract 5x from each side

5x+28 -5x = 7x-4-5x

28 = 2x-4

Add 4 to each side

28+4 = 2x-4+4

32 = 2x

Divide by 2

32/2 = 2x/2

16 =x

Step-by-step explanation:

(5x+28)=(7x-4)

2x=32

x=16

Which is the equation of a parabola with Vertex (0,0) and focus (0, 2)?
a. ya = 8x
c. x2 = 8
b. y2 = 4x
d. x2 = 4y

Answers

Answer:

So the equation of the parabola is x2=8y i don't really know

191+13=13+191191+13=13+191
what type of property is that

Answers

what nice question that we can understand ok

I’m not going to lie I don’t know what’s going on with this question

Question 11 of 25
If the point (1, 4) is on the graph of an equation, which statement
must be true?
A. There are solutions to the equation for the values x = 1 and
x = 4.
B. The values x = 1 and y = 4 are the only values that make the
equation true.
C. The values x = 4 and y= 1 make the equation true.
D. The values x = 1 and y = 4 make the equation true.
SUBMIT

Answers

Answer:

D

Step-by-step explanation:

simple : when we have a point defined as (1, 4), it means x = 1, y = 4.

and since the point is on the graph of a function/equation it means that when we use x = 1 and calculate the equation, we get 4 as result (= y). so yes, that means that both sides of the "=" sign are indeed equal for this pair of values, which makes the equation true.

but there will be usually many other pairs that do that too.

Enter the missing values in the area model for multiplication below.
5 x 35
30
+
5
25
try

Answers

The box on the left would be 150
The box on the right would be 5

find the missing length indicated​

Answers

Answer:

192

Step-by-step explanation:

Apply the geometric mean formula to solve for x, which is the altitude of the right triangle.

The formula is:

h = √(mn)

h = x = ?

m = 144

n = 400 - 144 = 256

Substitute

h = √(256*144)

h = √36,864

h = 192

Therefore, x = 192

Write an algebraic expression for “three less than the sum of a number and a four”

Answers

Answer: 3 < x+4

Explanation:

Answer:

(x + 4) - 3

Step-by-step explanation:

Let the number be x

(x +4) - 3

four less than the product of a number and 7 is eight more than that number

Answers

Answer:

2

Step-by-step explanation:

Replace the number with x, then the equation would be:

7x-4=x+8

7x-x=8+4

6x=12

6x/6=12/6

x=2

Solve for x.
5x + 3 – 2x = 24
X = [?]

Answers

[tex]\\ \sf \longmapsto 5x+3-2x=24[/tex]

[tex]\\ \sf \longmapsto 5x-2x+3=24[/tex]

[tex]\\ \sf \longmapsto 3x+3=24[/tex]

[tex]\\ \sf \longmapsto 3x=24-3[/tex]

[tex]\\ \sf \longmapsto 3x=21[/tex]

[tex]\\ \sf \longmapsto x=\dfrac{21}{3}[/tex]

[tex]\\ \sf \longmapsto x=7[/tex]

Answer:

x = [7]

Step-by-step explanation:

5x + 3 - 2x = 24

Combine like terms

5x - 2x + 3 = 24

Subtract 5x - 2x

3x + 3 = 24

Subtract 3 from both sides

3x = 21

Isolate the variable

x = 21/3

x = 7

in the triangle below which of the following best describes dh​

Answers

looks like an angle bisector to me

I am not sure. Is this right?

Answers

Answer:

14.4

Step-by-step explanation:

since the longest side is 24,find the shortest side

√20^2-12^2

√400-144

√256

=16

which means the other shortest side is 24-16

which is 8

then you have to use the 12 and 8 to find the unknown side

√12^2+8^2

144+64

√208

14.4

I hope this helps

Pls help if u only know the correct answer!! Thanks!! :))

Answers

Answer:

3.5, 4.2

Step-by-step explanation:

reflect make it not negative and its on y axis so y not affected

If the order of two matrices are different, then the addition of matrices does not exists true or false

Answers

Answer:

true

Step-by-step explanation:

A matrix can only be added/subtracted with another matrix if the two matrices have the same dimensions

Solve for X. Geometry

Answers

Answer:

x=12

Step-by-step explanation:

QS=QR+RS

2x-11=7+x-6

x=12

If the equation of the line is
(5y + 1)/10 = x, then what is the slope of the line?

A. 10
B. 5
C. 2
D. -1

Answers

Answer:

2

Step-by-step explanation:

(5y + 1)/10 = x

Multiply each side by 10

(5y + 1)/10*10 =10 x

(5y + 1) = 10x

Subtract 1

5y +1-1 = 10x-1

5y = 10x-1

Divide by 5

5y/5 = 10x/5 - 1/5

y = 2x-1/5

This is in slope intercept form y = mx+b where m is the slope and b is the y intercept

The slope is 2

Find the number in which 9 has greater value.
0.5689
5.6890
56.89
569.80

Answers

Answer:

569.80

Step-by-step explanation:

Among all the choices, the digit 9 has the greatest value in number 569.80. For the reason that 9 got ones value - which are greatest than other.

[tex]\tt{ \green{P} \orange{s} \red{y} \blue{x} \pink{c} \purple{h} \green{i} e}[/tex]

can someone do these easy questions. I forgot fractions​

Answers

1. 3/4
2. 3/4
3. 3 whole parts one missing part
4. 0.75 75% 6/8
5. It’s closer to 1 on a number line because it’s more then 1/2
6. A fraction greater could be 7/8 Bc it’s more then 6/8 and 6/8 is equal to 3/4
A fraction less then could be 1/4 Bc 1 is less then 3

Answer:

3/4 as in three forth shaded in

75%

1/4 missing blank

closer to whole

1/4 is what is missing that is less than 3/4 which is what it is your welcome.

A. Explain why the point (100,2) is on the graph.
B. What is the x-intercept of the graph? Explain how you know.
49
C. When will the graph meet the line y = 5? Explain how you know.

Answers

Answer:

the log function is the "inverse" function of an exponential function

by definition  [tex]log_{a} b = c[/tex]  then [tex]a^{c} = b[/tex]

in this problem you have [tex]log_{10} 100[/tex]

thus what x solves this ?  [tex]10^{x} = 100[/tex]  the answer is [tex]10^{2}[/tex]

thus (100,2)

B) the x intercept is when y = 0

   [tex]10^{0} = 1[/tex]

   x intercept at (1,0)

C) at 100, the curve will hit y = 5000

Step-by-step explanation:

Solve for x: 2(3x + 4) - 3(x - 1) = x - 1  ​

Answers

Answer:

x=-113

Step-by-step explanation:

2(3x+4)−3(x−1)=x−1x

Step 1: Simplify both sides of the equation.

2(3x+4)−3(x−1)=x−1x

(2)(3x)+(2)(4)+(−3)(x)+(−3)(−1)=x+−1x(Distribute)

6x+8+−3x+3=x+−x

(6x+−3x)+(8+3)=(x+−x)(Combine Like Terms)

3x+11=0

3x+11=0

Step 2: Subtract 11 from both sides.

3x+11−11=0−11

3x=−11

Step 3: Divide both sides by 3.

3x3=−113

x=−113

Answer:

x=−113

The answer is X= -113

If the odds against Deborahs winning first prize in a chess tournament are 1 to 11 what is the probability of the event that she will win first prize

Answers

Answer: [tex]\dfrac{11}{12}[/tex]

Step-by-step explanation:

Given

The odds against winning in a chess tournament are 1 to 11.

Odds is defined as the ratio of the probability of occurrence to the non-occurrence of event.

[tex]\therefore \text{Probability that event will occur is P'=}\dfrac{1}{1+11}\\\\\Rightarrow P'=\dfrac{1}{12}[/tex]

Probability of non-occurrence i.e. she wins the first prize is

[tex]\Rightarrow P=1-\dfrac{1}{12}\\\\\Rightarrow P=\dfrac{11}{12}[/tex]

True or False: The function y = (x2 – 4)/(x2 – 4) is continuous for all values of x

Answers

Answer:

False

Step-by-step explanation:

The function isn't continuous at x=±2 as a function isn't continuous at the point where the function becomes undefined, this function becomes undefined when the denominator is equal to 0. So x^2-4=0 => x=±2.

Find the nth term of the arithmetic
sequence - 1,2,5, ....

A. 3n - 2
B. -2n + 1
C. 2n + 2
D. 3n - 4

Answers

Answer:

3n -4

Step-by-step explanation:

We are adding 3 each time

-1+3 =2

2+3 = 5

The formula for an arithmetic sequence is

an = a1+d(n-1) where a1 is the first term and d is the common difference

an = -1+3(n-1)

    = -1 +3n -3

    = 3n -4

Help anyone can help me do the question,I will mark brainlest.​

Answers

Step-by-step explanation:

Ans 6 Use Pythagoras theorem...

Hypotneuse²=Base²+Height²

6²=4²+H²

36-16=H²

20=H²

√20=H

√2×2×5=H

2√5 = H....

hope it helps

Problem 4

Draw a diagonal to connect the opposite non-right angles as shown below. This forms two right triangles. As you can probably guess, we'll use the pythagorean theorem to find the length of this segment. We'll call it x.

a^2+b^2 = c^2

7^2+24^2 = x^2

49+576 = x^2

x^2 = 625

x = sqrt(625)

x = 25

The hypotenuse of each right triangle is 25 units long.

Move your focus to the upper right triangle. We can use the pythagorean theorem again in a slightly different way to find y.

a^2+b^2 = c^2

15^2+y^2 = 25^2

225+y^2 = 625

y^2 = 625-225

y^2 = 400

y = sqrt(400)

y = 20

Answer:  y = 20

====================================================

Problem 6

The largest triangle, ignore segment x for now, is isosceles because we have two congruent sides.

As with any isosceles triangle, the vertex angle is bisected by the perpendicular bisector as shown in the diagram. This produces two identical right triangles. Most importantly, it means that horizontal side 4 is cut in half to 4/2 = 2 units.

In short, each smaller right triangle has a horizontal leg of 2, vertical leg x, and hypotenuse 6. Let's focus on just one of those right triangles.

Like the previous problem, we'll use the pythagorean theorem to find x.

a^2+b^2 = c^2

x^2+2^2 = 6^2

x^2+4 = 36

x^2 = 36-4

x^2 = 32

x = sqrt(32)

x = sqrt(16*2)

x = sqrt(16)*sqrt(2)

x = 4*sqrt(2)

Answer:  x = 4*sqrt(2)

8TH GRADE MATH ¯\_(ツ)_/¯

What is the fractional equivalent of the repeating decimal n = 0.1515.... ?

Answer the questions to find out.

1. How many repeating digits does the number represented by n have?

2. You need to multiply n by a power of 10 to help you find the fraction. Decide on the power of 10 to multiply by, and tell how you identified that number.

3. Write an equation where the left side is your power of 10 time n and the right side is the result of multiplying 0.1515... by that power.

4. Write the original equation, n = 0.1515... underneath your equation from question 3. Then subtract the equations. Show your work!

5. Write n as a fraction in simplest form. Show your work!​

Answers

If n = 0.151515…, then 100n = 15.151515…

Then

100n - n = 15.151515… - 0.151515…

99n = 15

n = 15/99 = 5/33

4 is a square number and also an even number. How many other whole numbers less than 50 are an even, square number?

Answers

heres a picture bc im too lazy to write it


Solve for x in the drawing above.

Answers

Answer:

x is 28°

Step-by-step explanation:

Alternate angles of parallel lines:

[tex]{ \sf{x = \frac{1}{2} \times 56 \degree }} \\ { \sf{x = 28 \degree}}[/tex]

Answer:

Hello,

28°

Step-by-step explanation:

Calculate the complementary angle of x =y

In the triangle on the right :

y+y+56°=180°

y=62°

So x=90°-62°=28°

Other Questions
I cant solve plz help me ... 35. When did you start loving him?A. How long has you loved him?B. How long hasnt you loved him?C. How long havent you loved him?D. How long have you loved him? In a food preference experiment, 80 lizards were given the opportunity to choose to eat one of three different species of insects. The results showed that 33 of the lizards chose species A, 12 chose species B, and 35 chose species C. They conducted a Chi-squared analysis to test for equal preference. What are the Null and Alternate hypothesis for this test The number of solutions of |x - 1| + |x - 3| = 2 is The slope of a line is -4 and Y negative intercept is -3 what is the equation of the line written in slope intercept form A coffee pot holds 3 3/4 quarts of coffee. How much is this in cups. which statement most accurately describes a major criticism of the electoral college system?A. it is tied too closely to the popular vote in each stateB.it requires candidates to campaign throughout the entire countryC.it allows for faithless electors who do not vote according to the wishes of their states D.it undermines federalism by taking power away from the states The measure of one of the small angles of a right triangle is 45 less than twice the measure of the other small angle. Find the measure of both angles. 3. Grace has twice as much money as tadTogether they have $36. How much Moneydes Tad have? Shelton Co. purchased a parcel of land six years ago for $860,500. At that time, the firm invested $132,000 in grading the site so that it would be usable. Since the firm wasn't ready to use the site itself at that time, it decided to lease the land for $47,500 a year. The company is now considering building a warehouse on the site as the rental lease is expiring. The current value of the land is $912,000. What value should be included in the initial cost of the warehouse project for the use of this land find the value of trigonometric ratio What is the percent increase each year for the investment? This is for my math class Please help me , I also have to show work on paper Determine the highest level of protein structure described by each item. a. Primary structure b. Secondary structure c. Tertiary structure d. Quaternary structure1. order of amino acids 2. overall macromolecule structure containing more than one polypeptide chain Will Mark Brainlest Help Please ,,,, find the value of x and y You have 2 5 sided dice, what's the probability the addition of rolling both Where is the blue dot on the number line? (please hurry!) Peyton's management professor just told her class that the final exam is optional for students like Peyton who currently have an A. Peyton decides to take the exam anyway because she likes the subject and wants to master the material. Peyton is motivated to take the exam by a(n) Group of answer choices extrinsic reward. physiological need. high need for power. hygiene factor. intrinsic reward. Does a rhetorical question need an answer?A. Yes, a long answerB. NoC. Yes, followed by another questionD. Yes, a short one